Convergence in distribution of minimum of IID random variables












2












$begingroup$


I'm stuck on the following problem and could use a hint:




Let $Z_1,ldots,Z_n$ be IID random variables with density $f$. Suppose that $mathbb{P}(Z_i > 0) = 1$ and that $lambda = lim_{x to 0^{+} } f(x) > 0$. Let:



$$X_n = nminlbrace Z_1,ldots,Z_nrbrace$$



Show that $X_n rightsquigarrow Z$ (converges in distribution) where $Z$ has an exponential distribution with mean $frac{1}{lambda}$.




So far I've figured out that:



$$
begin{align*}
mathbb{P}(X_n leq x) &= mathbb{P}(nminlbrace Z_1,ldots,Z_nrbrace leq x) \
&= mathbb{P}(Z_1 leq x/n cup ldots cup Z_n leq x/n) \
&= 1 - mathbb{P}(Z_1 > x/n cap ldots cap Z_n > x/n) \
&= 1 - mathbb{P}(Z_1 > x/n) cdot ldots cdot mathbb{P}(Z_n > x/n) \
&= 1 - mathbb{P}(Z_1 > x/n)^n \
&= 1 - (1-mathbb{P}(Z_1 leq x/n))^n \
&= 1 - (1-F_{Z_1}(x/n))^n
end{align*}
$$



But I'm not sure where to go from here. Although I know the CDF/PDF for $Z$, I don't know it for $Z_1,ldots,Z_n$, and it's not immediately obvious to me how I can leverage the fact that $Z_1,ldots,Z_n$ are guaranteed to be positive, other than if I rewrite $F_{Z_1}(x/n)$ to be an explicit integral of $f$ that the lower bound of the integral can be zero instead of negative infinity.



If I just try to take the limit as $n$ goes to infinity of both sides I can't simplify the RHS expression any further because of the power of $n$. I know that $a_n to a implies (1 + frac{a_n}{n})^n to e^a$ but I haven't managed to successfully apply it -- I assume this somehow helps me collapse the RHS into the CDF for an exponential, because I don't see another way to get $e$.



Finally I don't have any idea how to leverage the definition of lambda. Usually we care about right continuity of the CDF not the density function.










share|cite|improve this question











$endgroup$












  • $begingroup$
    Consider the Taylor series of $F_{Z_1}$ at x=0. Also, I suspect that you have gotten your X's and Z's mixed up some places.
    $endgroup$
    – Mike Hawk
    Dec 16 '18 at 22:28










  • $begingroup$
    @MikeHawk You were right about mixing up, copy and pasted a latex template and modified it and made a few mistakes. Hopefully fixed now...
    $endgroup$
    – Joseph Garvin
    Dec 16 '18 at 22:42










  • $begingroup$
    @MikeHawk: I don't see how I can use that Taylor series, because I don't know the CDF for $Z_1$, only for $Z$.
    $endgroup$
    – Joseph Garvin
    Jan 8 at 23:06










  • $begingroup$
    But you know $F_{Z_1}'(0)=lambda$, so you just have to show that $(1-lambda x/n+O(1/n^2))^nto e^{-lambda x}$
    $endgroup$
    – Mike Hawk
    Jan 9 at 16:45
















2












$begingroup$


I'm stuck on the following problem and could use a hint:




Let $Z_1,ldots,Z_n$ be IID random variables with density $f$. Suppose that $mathbb{P}(Z_i > 0) = 1$ and that $lambda = lim_{x to 0^{+} } f(x) > 0$. Let:



$$X_n = nminlbrace Z_1,ldots,Z_nrbrace$$



Show that $X_n rightsquigarrow Z$ (converges in distribution) where $Z$ has an exponential distribution with mean $frac{1}{lambda}$.




So far I've figured out that:



$$
begin{align*}
mathbb{P}(X_n leq x) &= mathbb{P}(nminlbrace Z_1,ldots,Z_nrbrace leq x) \
&= mathbb{P}(Z_1 leq x/n cup ldots cup Z_n leq x/n) \
&= 1 - mathbb{P}(Z_1 > x/n cap ldots cap Z_n > x/n) \
&= 1 - mathbb{P}(Z_1 > x/n) cdot ldots cdot mathbb{P}(Z_n > x/n) \
&= 1 - mathbb{P}(Z_1 > x/n)^n \
&= 1 - (1-mathbb{P}(Z_1 leq x/n))^n \
&= 1 - (1-F_{Z_1}(x/n))^n
end{align*}
$$



But I'm not sure where to go from here. Although I know the CDF/PDF for $Z$, I don't know it for $Z_1,ldots,Z_n$, and it's not immediately obvious to me how I can leverage the fact that $Z_1,ldots,Z_n$ are guaranteed to be positive, other than if I rewrite $F_{Z_1}(x/n)$ to be an explicit integral of $f$ that the lower bound of the integral can be zero instead of negative infinity.



If I just try to take the limit as $n$ goes to infinity of both sides I can't simplify the RHS expression any further because of the power of $n$. I know that $a_n to a implies (1 + frac{a_n}{n})^n to e^a$ but I haven't managed to successfully apply it -- I assume this somehow helps me collapse the RHS into the CDF for an exponential, because I don't see another way to get $e$.



Finally I don't have any idea how to leverage the definition of lambda. Usually we care about right continuity of the CDF not the density function.










share|cite|improve this question











$endgroup$












  • $begingroup$
    Consider the Taylor series of $F_{Z_1}$ at x=0. Also, I suspect that you have gotten your X's and Z's mixed up some places.
    $endgroup$
    – Mike Hawk
    Dec 16 '18 at 22:28










  • $begingroup$
    @MikeHawk You were right about mixing up, copy and pasted a latex template and modified it and made a few mistakes. Hopefully fixed now...
    $endgroup$
    – Joseph Garvin
    Dec 16 '18 at 22:42










  • $begingroup$
    @MikeHawk: I don't see how I can use that Taylor series, because I don't know the CDF for $Z_1$, only for $Z$.
    $endgroup$
    – Joseph Garvin
    Jan 8 at 23:06










  • $begingroup$
    But you know $F_{Z_1}'(0)=lambda$, so you just have to show that $(1-lambda x/n+O(1/n^2))^nto e^{-lambda x}$
    $endgroup$
    – Mike Hawk
    Jan 9 at 16:45














2












2








2





$begingroup$


I'm stuck on the following problem and could use a hint:




Let $Z_1,ldots,Z_n$ be IID random variables with density $f$. Suppose that $mathbb{P}(Z_i > 0) = 1$ and that $lambda = lim_{x to 0^{+} } f(x) > 0$. Let:



$$X_n = nminlbrace Z_1,ldots,Z_nrbrace$$



Show that $X_n rightsquigarrow Z$ (converges in distribution) where $Z$ has an exponential distribution with mean $frac{1}{lambda}$.




So far I've figured out that:



$$
begin{align*}
mathbb{P}(X_n leq x) &= mathbb{P}(nminlbrace Z_1,ldots,Z_nrbrace leq x) \
&= mathbb{P}(Z_1 leq x/n cup ldots cup Z_n leq x/n) \
&= 1 - mathbb{P}(Z_1 > x/n cap ldots cap Z_n > x/n) \
&= 1 - mathbb{P}(Z_1 > x/n) cdot ldots cdot mathbb{P}(Z_n > x/n) \
&= 1 - mathbb{P}(Z_1 > x/n)^n \
&= 1 - (1-mathbb{P}(Z_1 leq x/n))^n \
&= 1 - (1-F_{Z_1}(x/n))^n
end{align*}
$$



But I'm not sure where to go from here. Although I know the CDF/PDF for $Z$, I don't know it for $Z_1,ldots,Z_n$, and it's not immediately obvious to me how I can leverage the fact that $Z_1,ldots,Z_n$ are guaranteed to be positive, other than if I rewrite $F_{Z_1}(x/n)$ to be an explicit integral of $f$ that the lower bound of the integral can be zero instead of negative infinity.



If I just try to take the limit as $n$ goes to infinity of both sides I can't simplify the RHS expression any further because of the power of $n$. I know that $a_n to a implies (1 + frac{a_n}{n})^n to e^a$ but I haven't managed to successfully apply it -- I assume this somehow helps me collapse the RHS into the CDF for an exponential, because I don't see another way to get $e$.



Finally I don't have any idea how to leverage the definition of lambda. Usually we care about right continuity of the CDF not the density function.










share|cite|improve this question











$endgroup$




I'm stuck on the following problem and could use a hint:




Let $Z_1,ldots,Z_n$ be IID random variables with density $f$. Suppose that $mathbb{P}(Z_i > 0) = 1$ and that $lambda = lim_{x to 0^{+} } f(x) > 0$. Let:



$$X_n = nminlbrace Z_1,ldots,Z_nrbrace$$



Show that $X_n rightsquigarrow Z$ (converges in distribution) where $Z$ has an exponential distribution with mean $frac{1}{lambda}$.




So far I've figured out that:



$$
begin{align*}
mathbb{P}(X_n leq x) &= mathbb{P}(nminlbrace Z_1,ldots,Z_nrbrace leq x) \
&= mathbb{P}(Z_1 leq x/n cup ldots cup Z_n leq x/n) \
&= 1 - mathbb{P}(Z_1 > x/n cap ldots cap Z_n > x/n) \
&= 1 - mathbb{P}(Z_1 > x/n) cdot ldots cdot mathbb{P}(Z_n > x/n) \
&= 1 - mathbb{P}(Z_1 > x/n)^n \
&= 1 - (1-mathbb{P}(Z_1 leq x/n))^n \
&= 1 - (1-F_{Z_1}(x/n))^n
end{align*}
$$



But I'm not sure where to go from here. Although I know the CDF/PDF for $Z$, I don't know it for $Z_1,ldots,Z_n$, and it's not immediately obvious to me how I can leverage the fact that $Z_1,ldots,Z_n$ are guaranteed to be positive, other than if I rewrite $F_{Z_1}(x/n)$ to be an explicit integral of $f$ that the lower bound of the integral can be zero instead of negative infinity.



If I just try to take the limit as $n$ goes to infinity of both sides I can't simplify the RHS expression any further because of the power of $n$. I know that $a_n to a implies (1 + frac{a_n}{n})^n to e^a$ but I haven't managed to successfully apply it -- I assume this somehow helps me collapse the RHS into the CDF for an exponential, because I don't see another way to get $e$.



Finally I don't have any idea how to leverage the definition of lambda. Usually we care about right continuity of the CDF not the density function.







calculus probability convergence exponential-distribution






share|cite|improve this question















share|cite|improve this question













share|cite|improve this question




share|cite|improve this question








edited Dec 16 '18 at 22:40







Joseph Garvin

















asked Dec 16 '18 at 22:22









Joseph GarvinJoseph Garvin

44928




44928












  • $begingroup$
    Consider the Taylor series of $F_{Z_1}$ at x=0. Also, I suspect that you have gotten your X's and Z's mixed up some places.
    $endgroup$
    – Mike Hawk
    Dec 16 '18 at 22:28










  • $begingroup$
    @MikeHawk You were right about mixing up, copy and pasted a latex template and modified it and made a few mistakes. Hopefully fixed now...
    $endgroup$
    – Joseph Garvin
    Dec 16 '18 at 22:42










  • $begingroup$
    @MikeHawk: I don't see how I can use that Taylor series, because I don't know the CDF for $Z_1$, only for $Z$.
    $endgroup$
    – Joseph Garvin
    Jan 8 at 23:06










  • $begingroup$
    But you know $F_{Z_1}'(0)=lambda$, so you just have to show that $(1-lambda x/n+O(1/n^2))^nto e^{-lambda x}$
    $endgroup$
    – Mike Hawk
    Jan 9 at 16:45


















  • $begingroup$
    Consider the Taylor series of $F_{Z_1}$ at x=0. Also, I suspect that you have gotten your X's and Z's mixed up some places.
    $endgroup$
    – Mike Hawk
    Dec 16 '18 at 22:28










  • $begingroup$
    @MikeHawk You were right about mixing up, copy and pasted a latex template and modified it and made a few mistakes. Hopefully fixed now...
    $endgroup$
    – Joseph Garvin
    Dec 16 '18 at 22:42










  • $begingroup$
    @MikeHawk: I don't see how I can use that Taylor series, because I don't know the CDF for $Z_1$, only for $Z$.
    $endgroup$
    – Joseph Garvin
    Jan 8 at 23:06










  • $begingroup$
    But you know $F_{Z_1}'(0)=lambda$, so you just have to show that $(1-lambda x/n+O(1/n^2))^nto e^{-lambda x}$
    $endgroup$
    – Mike Hawk
    Jan 9 at 16:45
















$begingroup$
Consider the Taylor series of $F_{Z_1}$ at x=0. Also, I suspect that you have gotten your X's and Z's mixed up some places.
$endgroup$
– Mike Hawk
Dec 16 '18 at 22:28




$begingroup$
Consider the Taylor series of $F_{Z_1}$ at x=0. Also, I suspect that you have gotten your X's and Z's mixed up some places.
$endgroup$
– Mike Hawk
Dec 16 '18 at 22:28












$begingroup$
@MikeHawk You were right about mixing up, copy and pasted a latex template and modified it and made a few mistakes. Hopefully fixed now...
$endgroup$
– Joseph Garvin
Dec 16 '18 at 22:42




$begingroup$
@MikeHawk You were right about mixing up, copy and pasted a latex template and modified it and made a few mistakes. Hopefully fixed now...
$endgroup$
– Joseph Garvin
Dec 16 '18 at 22:42












$begingroup$
@MikeHawk: I don't see how I can use that Taylor series, because I don't know the CDF for $Z_1$, only for $Z$.
$endgroup$
– Joseph Garvin
Jan 8 at 23:06




$begingroup$
@MikeHawk: I don't see how I can use that Taylor series, because I don't know the CDF for $Z_1$, only for $Z$.
$endgroup$
– Joseph Garvin
Jan 8 at 23:06












$begingroup$
But you know $F_{Z_1}'(0)=lambda$, so you just have to show that $(1-lambda x/n+O(1/n^2))^nto e^{-lambda x}$
$endgroup$
– Mike Hawk
Jan 9 at 16:45




$begingroup$
But you know $F_{Z_1}'(0)=lambda$, so you just have to show that $(1-lambda x/n+O(1/n^2))^nto e^{-lambda x}$
$endgroup$
– Mike Hawk
Jan 9 at 16:45










1 Answer
1






active

oldest

votes


















0












$begingroup$

You need to prove this probability approximates $1-exp -lambda x$, i.e. that $(1-F_{Z_i}(x/n))^napproxexp -lambda x$. But for $ngg x$, $$F_{Z_i}(0)=0implies F_{Z_i}(x/n)sim frac{x}{n} F_{Z_i}'(0)=frac{lambda x}{n},$$so the claim follows from $exp -t=lim_{ntoinfty}(1-frac{t}{n})^n$.






share|cite|improve this answer









$endgroup$













  • $begingroup$
    what does the n >> x notation mean?
    $endgroup$
    – Joseph Garvin
    Jan 6 at 18:59






  • 1




    $begingroup$
    @JosephGarvin That $n$ is much greater than $x$.
    $endgroup$
    – J.G.
    Jan 6 at 19:20










  • $begingroup$
    what does the $sim$ notation mean?
    $endgroup$
    – Joseph Garvin
    Jan 6 at 21:54












  • $begingroup$
    @JosephGarvin is asymptotic to.
    $endgroup$
    – J.G.
    Jan 6 at 21:55










  • $begingroup$
    I don't understand how you conclude $ F_{Z_i}(x/n)sim frac{x}{n} F_{Z_i}'(0)$ ? how're those expressions related?
    $endgroup$
    – Joseph Garvin
    Jan 9 at 0:00











Your Answer





StackExchange.ifUsing("editor", function () {
return StackExchange.using("mathjaxEditing", function () {
StackExchange.MarkdownEditor.creationCallbacks.add(function (editor, postfix) {
StackExchange.mathjaxEditing.prepareWmdForMathJax(editor, postfix, [["$", "$"], ["\\(","\\)"]]);
});
});
}, "mathjax-editing");

StackExchange.ready(function() {
var channelOptions = {
tags: "".split(" "),
id: "69"
};
initTagRenderer("".split(" "), "".split(" "), channelOptions);

StackExchange.using("externalEditor", function() {
// Have to fire editor after snippets, if snippets enabled
if (StackExchange.settings.snippets.snippetsEnabled) {
StackExchange.using("snippets", function() {
createEditor();
});
}
else {
createEditor();
}
});

function createEditor() {
StackExchange.prepareEditor({
heartbeatType: 'answer',
autoActivateHeartbeat: false,
convertImagesToLinks: true,
noModals: true,
showLowRepImageUploadWarning: true,
reputationToPostImages: 10,
bindNavPrevention: true,
postfix: "",
imageUploader: {
brandingHtml: "Powered by u003ca class="icon-imgur-white" href="https://imgur.com/"u003eu003c/au003e",
contentPolicyHtml: "User contributions licensed under u003ca href="https://creativecommons.org/licenses/by-sa/3.0/"u003ecc by-sa 3.0 with attribution requiredu003c/au003e u003ca href="https://stackoverflow.com/legal/content-policy"u003e(content policy)u003c/au003e",
allowUrls: true
},
noCode: true, onDemand: true,
discardSelector: ".discard-answer"
,immediatelyShowMarkdownHelp:true
});


}
});














draft saved

draft discarded


















StackExchange.ready(
function () {
StackExchange.openid.initPostLogin('.new-post-login', 'https%3a%2f%2fmath.stackexchange.com%2fquestions%2f3043257%2fconvergence-in-distribution-of-minimum-of-iid-random-variables%23new-answer', 'question_page');
}
);

Post as a guest















Required, but never shown

























1 Answer
1






active

oldest

votes








1 Answer
1






active

oldest

votes









active

oldest

votes






active

oldest

votes









0












$begingroup$

You need to prove this probability approximates $1-exp -lambda x$, i.e. that $(1-F_{Z_i}(x/n))^napproxexp -lambda x$. But for $ngg x$, $$F_{Z_i}(0)=0implies F_{Z_i}(x/n)sim frac{x}{n} F_{Z_i}'(0)=frac{lambda x}{n},$$so the claim follows from $exp -t=lim_{ntoinfty}(1-frac{t}{n})^n$.






share|cite|improve this answer









$endgroup$













  • $begingroup$
    what does the n >> x notation mean?
    $endgroup$
    – Joseph Garvin
    Jan 6 at 18:59






  • 1




    $begingroup$
    @JosephGarvin That $n$ is much greater than $x$.
    $endgroup$
    – J.G.
    Jan 6 at 19:20










  • $begingroup$
    what does the $sim$ notation mean?
    $endgroup$
    – Joseph Garvin
    Jan 6 at 21:54












  • $begingroup$
    @JosephGarvin is asymptotic to.
    $endgroup$
    – J.G.
    Jan 6 at 21:55










  • $begingroup$
    I don't understand how you conclude $ F_{Z_i}(x/n)sim frac{x}{n} F_{Z_i}'(0)$ ? how're those expressions related?
    $endgroup$
    – Joseph Garvin
    Jan 9 at 0:00
















0












$begingroup$

You need to prove this probability approximates $1-exp -lambda x$, i.e. that $(1-F_{Z_i}(x/n))^napproxexp -lambda x$. But for $ngg x$, $$F_{Z_i}(0)=0implies F_{Z_i}(x/n)sim frac{x}{n} F_{Z_i}'(0)=frac{lambda x}{n},$$so the claim follows from $exp -t=lim_{ntoinfty}(1-frac{t}{n})^n$.






share|cite|improve this answer









$endgroup$













  • $begingroup$
    what does the n >> x notation mean?
    $endgroup$
    – Joseph Garvin
    Jan 6 at 18:59






  • 1




    $begingroup$
    @JosephGarvin That $n$ is much greater than $x$.
    $endgroup$
    – J.G.
    Jan 6 at 19:20










  • $begingroup$
    what does the $sim$ notation mean?
    $endgroup$
    – Joseph Garvin
    Jan 6 at 21:54












  • $begingroup$
    @JosephGarvin is asymptotic to.
    $endgroup$
    – J.G.
    Jan 6 at 21:55










  • $begingroup$
    I don't understand how you conclude $ F_{Z_i}(x/n)sim frac{x}{n} F_{Z_i}'(0)$ ? how're those expressions related?
    $endgroup$
    – Joseph Garvin
    Jan 9 at 0:00














0












0








0





$begingroup$

You need to prove this probability approximates $1-exp -lambda x$, i.e. that $(1-F_{Z_i}(x/n))^napproxexp -lambda x$. But for $ngg x$, $$F_{Z_i}(0)=0implies F_{Z_i}(x/n)sim frac{x}{n} F_{Z_i}'(0)=frac{lambda x}{n},$$so the claim follows from $exp -t=lim_{ntoinfty}(1-frac{t}{n})^n$.






share|cite|improve this answer









$endgroup$



You need to prove this probability approximates $1-exp -lambda x$, i.e. that $(1-F_{Z_i}(x/n))^napproxexp -lambda x$. But for $ngg x$, $$F_{Z_i}(0)=0implies F_{Z_i}(x/n)sim frac{x}{n} F_{Z_i}'(0)=frac{lambda x}{n},$$so the claim follows from $exp -t=lim_{ntoinfty}(1-frac{t}{n})^n$.







share|cite|improve this answer












share|cite|improve this answer



share|cite|improve this answer










answered Dec 16 '18 at 22:50









J.G.J.G.

26.3k22541




26.3k22541












  • $begingroup$
    what does the n >> x notation mean?
    $endgroup$
    – Joseph Garvin
    Jan 6 at 18:59






  • 1




    $begingroup$
    @JosephGarvin That $n$ is much greater than $x$.
    $endgroup$
    – J.G.
    Jan 6 at 19:20










  • $begingroup$
    what does the $sim$ notation mean?
    $endgroup$
    – Joseph Garvin
    Jan 6 at 21:54












  • $begingroup$
    @JosephGarvin is asymptotic to.
    $endgroup$
    – J.G.
    Jan 6 at 21:55










  • $begingroup$
    I don't understand how you conclude $ F_{Z_i}(x/n)sim frac{x}{n} F_{Z_i}'(0)$ ? how're those expressions related?
    $endgroup$
    – Joseph Garvin
    Jan 9 at 0:00


















  • $begingroup$
    what does the n >> x notation mean?
    $endgroup$
    – Joseph Garvin
    Jan 6 at 18:59






  • 1




    $begingroup$
    @JosephGarvin That $n$ is much greater than $x$.
    $endgroup$
    – J.G.
    Jan 6 at 19:20










  • $begingroup$
    what does the $sim$ notation mean?
    $endgroup$
    – Joseph Garvin
    Jan 6 at 21:54












  • $begingroup$
    @JosephGarvin is asymptotic to.
    $endgroup$
    – J.G.
    Jan 6 at 21:55










  • $begingroup$
    I don't understand how you conclude $ F_{Z_i}(x/n)sim frac{x}{n} F_{Z_i}'(0)$ ? how're those expressions related?
    $endgroup$
    – Joseph Garvin
    Jan 9 at 0:00
















$begingroup$
what does the n >> x notation mean?
$endgroup$
– Joseph Garvin
Jan 6 at 18:59




$begingroup$
what does the n >> x notation mean?
$endgroup$
– Joseph Garvin
Jan 6 at 18:59




1




1




$begingroup$
@JosephGarvin That $n$ is much greater than $x$.
$endgroup$
– J.G.
Jan 6 at 19:20




$begingroup$
@JosephGarvin That $n$ is much greater than $x$.
$endgroup$
– J.G.
Jan 6 at 19:20












$begingroup$
what does the $sim$ notation mean?
$endgroup$
– Joseph Garvin
Jan 6 at 21:54






$begingroup$
what does the $sim$ notation mean?
$endgroup$
– Joseph Garvin
Jan 6 at 21:54














$begingroup$
@JosephGarvin is asymptotic to.
$endgroup$
– J.G.
Jan 6 at 21:55




$begingroup$
@JosephGarvin is asymptotic to.
$endgroup$
– J.G.
Jan 6 at 21:55












$begingroup$
I don't understand how you conclude $ F_{Z_i}(x/n)sim frac{x}{n} F_{Z_i}'(0)$ ? how're those expressions related?
$endgroup$
– Joseph Garvin
Jan 9 at 0:00




$begingroup$
I don't understand how you conclude $ F_{Z_i}(x/n)sim frac{x}{n} F_{Z_i}'(0)$ ? how're those expressions related?
$endgroup$
– Joseph Garvin
Jan 9 at 0:00


















draft saved

draft discarded




















































Thanks for contributing an answer to Mathematics Stack Exchange!


  • Please be sure to answer the question. Provide details and share your research!

But avoid



  • Asking for help, clarification, or responding to other answers.

  • Making statements based on opinion; back them up with references or personal experience.


Use MathJax to format equations. MathJax reference.


To learn more, see our tips on writing great answers.




draft saved


draft discarded














StackExchange.ready(
function () {
StackExchange.openid.initPostLogin('.new-post-login', 'https%3a%2f%2fmath.stackexchange.com%2fquestions%2f3043257%2fconvergence-in-distribution-of-minimum-of-iid-random-variables%23new-answer', 'question_page');
}
);

Post as a guest















Required, but never shown





















































Required, but never shown














Required, but never shown












Required, but never shown







Required, but never shown

































Required, but never shown














Required, but never shown












Required, but never shown







Required, but never shown







Popular posts from this blog

Quarter-circle Tiles

build a pushdown automaton that recognizes the reverse language of a given pushdown automaton?

Mont Emei